udowodnij inkluzje

Zdania. Tautologie. Język matematyki. Wszelkie zagadnienia związane z logiką matematyczną...
piotrek2008
Użytkownik
Użytkownik
Posty: 69
Rejestracja: 17 paź 2008, o 09:19
Płeć: Mężczyzna
Lokalizacja: Warszawa
Podziękował: 3 razy

udowodnij inkluzje

Post autor: piotrek2008 »

Udowodnić inkluzje
1. \(\displaystyle{ (A \subset B) \wedge (C \subset D) \Rightarrow (A \vee C \subset B \vee D)}\)
2. \(\displaystyle{ (A \subset B) \Leftrightarrow }\)
3. \(\displaystyle{ (A \subset B) \Leftrightarrow [ (B \subset C) \Rightarrow [(C-A) \wedge (C-B)=C-B] ]}\)
4. \(\displaystyle{ [( \bigcap_{t \in T}A _{t} ) \vee ( \bigcap_{t \in T}B _{t} )]= \bigcap_{t, s \in T}(A _{t} \vee B _{s} ) \subset \bigcap_{t \in T}(A _{t} \vee B _{t} )}\)
Jan Kraszewski
Administrator
Administrator
Posty: 34128
Rejestracja: 20 mar 2006, o 21:54
Płeć: Mężczyzna
Lokalizacja: Wrocław
Podziękował: 3 razy
Pomógł: 5192 razy

udowodnij inkluzje

Post autor: Jan Kraszewski »

Ad 1. Po prawej stronie implikacji nie \(\displaystyle{ \lor}\) tylko \(\displaystyle{ \cup}\).
Ad 2. Coś ci zjadło na końcu...
Ad 3. Nie \(\displaystyle{ \land}\) tylko \(\displaystyle{ \cap}\).
Ad 4. To, co napisałeś, nie ma sensu.

JK
piotrek2008
Użytkownik
Użytkownik
Posty: 69
Rejestracja: 17 paź 2008, o 09:19
Płeć: Mężczyzna
Lokalizacja: Warszawa
Podziękował: 3 razy

udowodnij inkluzje

Post autor: piotrek2008 »

Nie wiedziałem jak napisać znak sumy i iloczynu w TeX i dlatego wcześniej tak napisałem .

Udowodnić inkluzje
a) \(\displaystyle{ (A \subset B)\cap(C \subset D) \Rightarrow (A\cup C \subset B\cup D)}\)
b) \(\displaystyle{ (A \subset B)\Leftrightarrow [(B \subset C) \Rightarrow [(C-A)\cap(C-B)=C-B]]}\)
c) \(\displaystyle{ A\cup(B-C)=[(A \cup B)-C]\cup(A \cap C)}\)
d) \(\displaystyle{ (A \subset B)\cap (C \subset D) \Rightarrow A-D \subset B-C}\)

Nie chodzi mi o to , żeby się rozpisywać , tylko o główne przejścia z jednej inkluzji w drugą.
Jan Kraszewski
Administrator
Administrator
Posty: 34128
Rejestracja: 20 mar 2006, o 21:54
Płeć: Mężczyzna
Lokalizacja: Wrocław
Podziękował: 3 razy
Pomógł: 5192 razy

udowodnij inkluzje

Post autor: Jan Kraszewski »

piotrek2008 pisze:a) \(\displaystyle{ (A \subset B)\cap(C \subset D) \Rightarrow (A\cup C \subset B\cup D)}\)
d) \(\displaystyle{ (A \subset B)\cap (C \subset D) \Rightarrow A-D \subset B-C}\)
Znów nie tak. Ma być:
a) \(\displaystyle{ (A B)\land C) D) A\cup C B\cup D}\)
d) \(\displaystyle{ (A B)\land (C D) A\setminus D B\setminus C}\)

A w kwestiach merytorycznych: z czym masz problem?
Podpunkt a) robi się wprost z def. zawierania i założeń.
Podpunkt d) tak samo, trzeba tylko pamiętać, że jeśli \(\displaystyle{ C \subseteq D}\), to dla dowolnego \(\displaystyle{ x}\) jeśli \(\displaystyle{ x\notin D}\), to \(\displaystyle{ x\notin C}\).
Podpunkt c) - pokazujesz, że oba zbiory mają te same elementy. Zacznij od \(\displaystyle{ x\in [(A\cup B)\setminus C]\cup (A\cap C)}\) i spróbuj dojść równoważnie do drugiej strony.
Podpunkt b) - ta równoważność nie zachodzi! Zachodzi tylko wynikanie\(\displaystyle{ \Rightarrow}\). Oznacza ono, że zakładając \(\displaystyle{ A\subseteq B}\) i \(\displaystyle{ B \subseteq C}\) pokazujesz, że \(\displaystyle{ (C\setminus A)\cap(C\setminus B)=C\setminus B}\). Tu najwygodniej skorzystać z twierdzenia \(\displaystyle{ X \subseteq Y \Leftrightarrow X\cap Y=X}\) i podpunktu d).
Wynikanie \(\displaystyle{ \Leftarrow}\) nie zachodzi. Dla \(\displaystyle{ A=\{1\},\ B=\{2\}, C=\emptyset}\) ta implikacja jest fałszywa.

JK
piotrek2008
Użytkownik
Użytkownik
Posty: 69
Rejestracja: 17 paź 2008, o 09:19
Płeć: Mężczyzna
Lokalizacja: Warszawa
Podziękował: 3 razy

udowodnij inkluzje

Post autor: piotrek2008 »

Nie wyszedł mi przykład d i c

d) \(\displaystyle{ (A \subset B)\land (C \subset D) \Rightarrow A\setminus D \subset B\setminus C}\)

założenie
1) \(\displaystyle{ x \in A \Rightarrow x \in B}\)
2)\(\displaystyle{ x \in C \Rightarrow x \in D}\) , a więc i \(\displaystyle{ x \notin D \Rightarrow x \notin C}\)

\(\displaystyle{ x \in A\setminus D \subset B\setminus C \Leftrightarrow x \in A \wedge x \notin D \Rightarrow x \in B \wedge x \notin D \Rightarrow x \in B \wedge x \notin C}\)
i dalej nie wiem jak to zapisać

c) \(\displaystyle{ A\cup(B\setminus C)=[(A \cup B)\setminus C]\cup(A \cap C)}\)
\(\displaystyle{ [(x \in A \vee x \in B) \wedge x \notin C] \vee ((x \in A \wedge x \in C)}\)


Mam jeszcze dwa dowody , które zrobiłem , tylko nie wiem , czy poprawnie.

a) \(\displaystyle{ \bigcup_{t \in T} \bigcup_{s \in S} {A}_{ts} = \bigcup_{s \in S} \bigcup_{t \in T} {A}_{ts}}\)

\(\displaystyle{ x\in \bigcup_{t \in T} \bigcup_{s \in S} A_{ts} \iff \exists_{t\in T} \exists_{s\in S} x\in A_{ts} \iff \exists_{s\in S} \exists_{t\in T} x\in A_{ts} \iff x\in \bigcup_{s \in S} \bigcup_{t \in T} {A}_{ts}}\)

b) \(\displaystyle{ \bigcup_{t\in T} {A}_{t} \cap \bigcup_{s\in S} {B}_{t}= \bigcup_{t\in T}\bigcup_{s\in S}( {A}_{t} \cap {B}_{s} )}\)

\(\displaystyle{ x\in \bigcup_{t\in T} {A}_{t} \cap \bigcup_{s\in S} {B}_{t} \iff \exists_{t\in T} x \in {A}_{t} \wedge \exists_{s\in S} x \in {B}_{t} \Leftarrow \exists_{t\in T} \exists_{s\in S} x \in ( {A}_{t} \wedge {B}_{s} \wedge s=t) \iff x \in \bigcup_{t\in T}\bigcup_{s\in S}( {A}_{t} \cap {B}_{s})}\)
Jan Kraszewski
Administrator
Administrator
Posty: 34128
Rejestracja: 20 mar 2006, o 21:54
Płeć: Mężczyzna
Lokalizacja: Wrocław
Podziękował: 3 razy
Pomógł: 5192 razy

udowodnij inkluzje

Post autor: Jan Kraszewski »

piotrek2008 pisze:d) \(\displaystyle{ (A \subset B)\land (C \subset D) \Rightarrow A\setminus D \subset B\setminus C}\)

założenie
1) \(\displaystyle{ x \in A \Rightarrow x \in B}\)
2)\(\displaystyle{ x \in C \Rightarrow x \in D}\) , a więc i \(\displaystyle{ x \notin D \Rightarrow x \notin C}\)

\(\displaystyle{ x \in A\setminus D \subset B\setminus C \Leftrightarrow x \in A \wedge x \notin D \Rightarrow x \in B \wedge x \notin D \Rightarrow x \in B \wedge x \notin C}\)
i dalej nie wiem jak to zapisać
Nie, nie, słuchaj, dowodzisz inkluzji. To nie polega na przekształcaniu napisów, tylko na zrozumieniu, co robisz. W tym przypadku (zgodnie z definicją inkluzji) masz pokazać, że jeśli \(\displaystyle{ x \in A\setminus D}\), to \(\displaystyle{ x \in B\setminus C}\). Masz zatem
\(\displaystyle{ x \in A\setminus D \Leftrightarrow x \in A \wedge x \notin D \Rightarrow x \in B \wedge x \notin D \Rightarrow x \in B \wedge x \notin C \Leftrightarrow x\in B\setminus C}\),
czyli to, co chciałeś.
piotrek2008 pisze:c) \(\displaystyle{ A\cup(B\setminus C)=[(A \cup B)\setminus C]\cup(A \cap C)}\)
\(\displaystyle{ [(x \in A \vee x \in B) \wedge x \notin C] \vee ((x \in A \wedge x \in C)}\)
Spróbuj najpierw skorzystać z rozdzielności koniunkcji względem alternatywy, potem z przemienności alternatywy, potem z potem jeszcze raz z rozdzielności koniunkcji względem alternatywy, ale "w drugą stronę" (czyli "wyłączyć przed nawias") i już prawie masz to, co trzeba.
piotrek2008 pisze:a) \(\displaystyle{ \bigcup_{t \in T} \bigcup_{s \in S} {A}_{ts} = \bigcup_{s \in S} \bigcup_{t \in T} {A}_{ts}}\)

\(\displaystyle{ x\in \bigcup_{t \in T} \bigcup_{s \in S} A_{ts} \iff \exists_{t\in T} \exists_{s\in S} x\in A_{ts} \iff \exists_{s\in S} \exists_{t\in T} x\in A_{ts} \iff x\in \bigcup_{s \in S} \bigcup_{t \in T} {A}_{ts}}\)
Dobrze.
piotrek2008 pisze:b) \(\displaystyle{ \bigcup_{t\in T} {A}_{t} \cap \bigcup_{s\in S} {B}_{t}= \bigcup_{t\in T}\bigcup_{s\in S}( {A}_{t} \cap {B}_{s} )}\)

\(\displaystyle{ x\in \bigcup_{t\in T} {A}_{t} \cap \bigcup_{s\in S} {B}_{t} \iff \exists_{t\in T} x \in {A}_{t} \wedge \exists_{s\in S} x \in {B}_{t} \Leftarrow \exists_{t\in T} \exists_{s\in S} x \in ( {A}_{t} \wedge {B}_{s} \wedge s=t) \iff x \in \bigcup_{t\in T}\bigcup_{s\in S}( {A}_{t} \cap {B}_{s})}\)
Zachachmęciłeś. Poza tym pomyłka w treści (w indeksie przy \(\displaystyle{ B}\) jest \(\displaystyle{ s}\)).
Powinno być
\(\displaystyle{ x\in \bigcup_{t\in T} {A}_{t} \cap \bigcup_{s\in S} {B}_{s} \iff \exists_{t\in T} x \in {A}_{t} \wedge \exists_{s\in S} x \in {B}_{t} \iff \\ \iff \exists_{t\in T} \exists_{s\in S}( x \in A_{t} \wedge x\in {B}_{s}) \iff \exists_{t\in T} \exists_{s\in S} x \in A_{t} \cap {B}_{s}\iff \\ \iff x \in \bigcup_{t\in T}\bigcup_{s\in S}( {A}_{t} \cap {B}_{s})}\)

JK
ODPOWIEDZ